Comparison | LSAT Logical Reasoning

แชร์
ฝัง
  • เผยแพร่เมื่อ 7 พ.ย. 2024

ความคิดเห็น • 17

  • @advaitarao-sharma6546
    @advaitarao-sharma6546 4 ปีที่แล้ว +6

    I have been struggling with LR; this framework is so good for me! Thank you!!!

    • @christiansamir1273
      @christiansamir1273 3 ปีที่แล้ว

      i dont mean to be offtopic but does anybody know of a method to log back into an instagram account..?
      I was stupid lost the login password. I appreciate any assistance you can offer me

  • @gxxl1122
    @gxxl1122 4 ปีที่แล้ว +4

    Best videos as usual, please please keep updating!

  • @Berrybrri
    @Berrybrri ปีที่แล้ว

    I like the guy’s personality in this. Makes it entertaining to study

  • @anir3162
    @anir3162 4 ปีที่แล้ว +2

    New subscriber to this channel because I love it that much! Adding these videos (and your channel) to my study routine!

    • @Chrystalxlopez
      @Chrystalxlopez 4 ปีที่แล้ว +1

      Hi there, what other resources have you found helpful? :) I'm struggling with games..

  • @dr.nancyd7007
    @dr.nancyd7007 4 ปีที่แล้ว +1

    Excellent presentation and explanations. Thank you!

  • @idealistthoughts3280
    @idealistthoughts3280 2 ปีที่แล้ว

    Great stuff. Went through all the LR videos, thanks so much.

  • @jakhongirabdurakhmonov7876
    @jakhongirabdurakhmonov7876 3 ปีที่แล้ว +1

    Very helpful and practical presentation I have ever experienced.

  • @jasonlou2013S
    @jasonlou2013S 2 ปีที่แล้ว

    12:59, the answer is D!

  • @seanroddy1922
    @seanroddy1922 ปีที่แล้ว

    great video

  • @L3gion3r
    @L3gion3r 4 ปีที่แล้ว +1

    I don't understand the Video Game Sales question at 18:38 . I don't see how E isn't out of scope, since, historically 3/4, of the purchases are made by 13-16yolds. To me it sounds like a fact: mainly 13-16 year olds purchase Vgames. E, in this case doesn't make sense to me as it directly contradicts this fact instead of weakening it. It speaks to me as "no, your stats are wrong author. the increase in sales the last 3 years is as a result of people over 16 years olds". It would make more sense to say that "even though the number of 13-16 yolds will decrease, this group will increase the number of purchases of Vgames in future", but such answer choice isn't available.

    • @ohthatpatrick949
      @ohthatpatrick949 4 ปีที่แล้ว +2

      I can see why it's a weird answer, since "Historically" seems to include the last 3 years, but "Historically" is an expression that reaches back way farther than just the last 3 years, so a trend that *on average* involves 13-16 year olds may nonetheless have recently shifted.
      If you think about changes wrought by climate change, it's natural to say "Historically, the sandpipers have returned to these beaches to make their nests, but over the last 3 years they have stopped coming".
      Also, not to freak you out, but there are a handful of Weaken questions in which the correct answer does seem to kind of contradict a premise (but like 5 max, in the history of questions). So you're right that it's not the typical feel, but give yourself permission to think "if this answer is true, as the question stem instructs me to accept, it definitely would have impact."

    • @carlosrodriguez9528
      @carlosrodriguez9528 3 ปีที่แล้ว +2

      The question is telling us to assume the answer choice will be true, hence “if true.” If it’s true that majority of video game buyers are over 16, then it weakens the argument that was made.

  • @sanskritibisht1356
    @sanskritibisht1356 ปีที่แล้ว

    15:17 / 26:22

  • @r.p.8906
    @r.p.8906 2 ปีที่แล้ว

    your page gives a 404 error.

    • @LSATLab
      @LSATLab  2 ปีที่แล้ว

      what page are you referring to? Something on TH-cam, or something on the actual LSAT Lab website?